不等式的证明之二

不等式的证明之二

  • 证明下述不等式
  • 证法一
  • 证法二
    • 证法二的补充

证明下述不等式

a , b , c a,b,c a,b,c 是正实数,请证明下述不等式:
11 a 5 a + 6 b + 11 b 5 b + 6 c + 11 c 5 c + 6 a ≤ 3 \begin{align} \sqrt{\frac{11a}{5a+6b}}+\sqrt{\frac{11b}{5b+6c}}+\sqrt{\frac{11c}{5c+6a}} \leq 3 \end{align} 5a+6b11a +5b+6c11b +5c+6a11c 3 上述不等式及其证明思路参考文献 [1],本文将给出更多的证明细节。
在证明之前,我们先回忆一下柯西·施瓦兹不等式的离散情形:

假设 a i , b i ∈ R a_i, b_i \in \mathbb{R} ai,biR i ∈ { 1 , 2 , 3 , ⋯ , n } i\in\{ 1,2,3,\cdots, n \} i{1,2,3,,n},则有
( ∑ i = 1 n a i b i ) 2 ≤ ∑ i = 1 n a i 2 ∑ i = 1 n b i 2 \begin{align} \left( \sum^{n}_{i=1} a_ib_i\right)^2 \leq \sum^{n}_{i=1}a_i^2 \sum^{n}_{i=1}b_i^2 \end{align} (i=1naibi)2i=1nai2i=1nbi2 等号成立当且仅当 a 1 / b 1 = a 2 / b 2 = ⋯ = a n / b n a_1/b_1=a_2/b_2=\cdots =a_n/b_n a1/b1=a2/b2==an/bn 成立或者 { a i } \{a_i\} {ai} { b i } \{b_i\} {bi} 中至少有一方全为0。

作为柯西·施瓦兹不等式的变形,我们有下面的等价形式:
∑ i = 1 n a i b i ≤ ∑ i = 1 n a i 2 ∑ i = 1 n b i 2 \begin{align} \sum^{n}_{i=1} a_ib_i \leq \sqrt{\sum^{n}_{i=1}a_i^2 \sum^{n}_{i=1}b_i^2} \end{align} i=1naibii=1nai2i=1nbi2 这个等价的柯西·施瓦兹不等式在下面会被用到。跟上一节内容相同, ∑ cyc \sum_{\text{cyc}} cyc 代表的是循环求和。

下面的证法一是较为简单的,证法二较为复杂,但更一般。读者如果觉得证法二比较难理解,就掌握证法一即可

证法一

注意到 ∑ cyc ( 5 c + 6 a ) = 11 ( a + b + c ) \sum_{\text{cyc}}(5c + 6a)=11(a+b+c) cyc(5c+6a)=11(a+b+c),以及 ∑ cyc a ( 5 b + 6 c ) = 11 ( a b + b c + c a ) \sum_{\text{cyc}} a(5b + 6c)=11(ab+bc+ca) cyca(5b+6c)=11(ab+bc+ca),为证明不等式(1),我们运用柯西·施瓦兹不等式:
11 a 5 a + 6 b + 11 b 5 b + 6 c + 11 c 5 c + 6 a = ∑ cyc 11 ( 5 c + 6 a ) ( 5 a + 6 b ) ( 5 b + 6 c ) ( 5 c + 6 a ) ⋅ a ( 5 b + 6 c ) ≤ ∑ cyc 11 ( 5 c + 6 a ) ( 5 a + 6 b ) ( 5 b + 6 c ) ( 5 c + 6 a ) ⋅ ∑ cyc a ( 5 b + 6 c ) ≤ 1 1 2 ( a + b + c ) ( 5 a + 6 b ) ( 5 b + 6 c ) ( 5 c + 6 a ) ⋅ ∑ cyc a ( 5 b + 6 c ) = 1 1 3 ( a + b + c ) ( a b + b c + c a ) ( 5 a + 6 b ) ( 5 b + 6 c ) ( 5 c + 6 a ) \begin{align} &\sqrt{\frac{11a}{5a+6b}}+\sqrt{\frac{11b}{5b+6c}}+\sqrt{\frac{11c}{5c+6a}} \\ &=\sum_{\text{cyc}}\sqrt{\frac {11(5c + 6a)} {(5a + 6b)(5b + 6c)(5c + 6a)} \cdot a(5b + 6c)} \\ &\leq \sqrt{\sum_{\text{cyc}}\frac {11(5c + 6a)} {(5a + 6b)(5b + 6c)(5c + 6a)} \cdot \sum_{\text{cyc}} a(5b + 6c) } \\ &\leq \sqrt{\frac {11^2 (a + b+ c)} {(5a + 6b)(5b + 6c)(5c + 6a)} \cdot \sum_{\text{cyc}} a(5b + 6c) } \\ &= \sqrt{\frac {11^3 (a + b+ c) (ab + bc + ca)} {(5a + 6b)(5b + 6c)(5c + 6a)}} \end{align} 5a+6b11a +5b+6c11b +5c+6a11c =cyc(5a+6b)(5b+6c)(5c+6a)11(5c+6a)a(5b+6c) cyc(5a+6b)(5b+6c)(5c+6a)11(5c+6a)cyca(5b+6c) (5a+6b)(5b+6c)(5c+6a)112(a+b+c)cyca(5b+6c) =(5a+6b)(5b+6c)(5c+6a)113(a+b+c)(ab+bc+ca) 为了求 1 1 3 ( a + b + c ) ( a b + b c + c a ) ( 5 a + 6 b ) ( 5 b + 6 c ) ( 5 c + 6 a ) \frac {11^3 (a + b+ c) (ab + bc + ca)} {(5a + 6b)(5b + 6c)(5c + 6a)} (5a+6b)(5b+6c)(5c+6a)113(a+b+c)(ab+bc+ca) 的上界,不妨令 1 1 3 ( a + b + c ) ( a b + b c + c a ) ( 5 a + 6 b ) ( 5 b + 6 c ) ( 5 c + 6 a ) ≤ k \frac {11^3 (a + b+ c) (ab + bc + ca)} {(5a + 6b)(5b + 6c)(5c + 6a)}\leq k (5a+6b)(5b+6c)(5c+6a)113(a+b+c)(ab+bc+ca)k,结合(1)式可猜测此处 k = 9 k=9 k=9,结合此假设则有:
1 1 3 ( a + b + c ) ( a b + b c + c a ) ≤ 9 ( 5 a + 6 b ) ( 5 b + 6 c ) ( 5 c + 6 a ) \begin{align} 11^3 (a + b+ c) (ab + bc + ca)\leq 9 (5a + 6b)(5b + 6c)(5c + 6a) \end{align} 113(a+b+c)(ab+bc+ca)9(5a+6b)(5b+6c)(5c+6a) 将(9)式改写为:
9 ( 5 a + 6 b ) ( 5 b + 6 c ) ( 5 c + 6 a ) − 1 1 3 ( a + b + c ) ( a b + b c + c a ) ≥ 0 \begin{align} 9 (5a + 6b)(5b + 6c)(5c + 6a)-11^3 (a + b+ c) (ab + bc + ca) \geq 0 \end{align} 9(5a+6b)(5b+6c)(5c+6a)113(a+b+c)(ab+bc+ca)0 下面我们来说明以上猜测的(10)式其实是正确的。首先由算术几何平均不等式我们有:
a b c = a b 2 ⋅ b c 2 ⋅ c a 2 3 ≤ 1 3 ( a b 2 + b c 2 + c a 2 ) a b c = a 2 b ⋅ b 2 c ⋅ c 2 a 3 ≤ 1 3 ( a 2 b + b 2 c + c 2 a ) \begin{align} abc = \sqrt[3]{ab^2\cdot bc^2 \cdot ca^2} \leq \frac 1 3 (ab^2 + bc^2 + ca^2) \\ abc= \sqrt[3]{a^2b\cdot b^2c \cdot c^2a}\leq \frac 1 3 (a^2b + b^2c + c^2a) \end{align} abc=3ab2bc2ca2 31(ab2+bc2+ca2)abc=3a2bb2cc2a 31(a2b+b2c+c2a) 结合(11)、(12)两式则有:
9 ( 5 a + 6 b ) ( 5 b + 6 c ) ( 5 c + 6 a ) − 1 1 3 ( a + b + c ) ( a b + b c + c a ) = 19 a 2 b + 289 a b 2 + 289 a 2 c − 924 a b c + 19 b 2 c + 19 a c 2 + 289 b c 2 = 289 ( a b 2 + b c 2 + c a 2 ) + 19 ( a 2 b + b 2 c + c 2 a ) − 924 a b c = 289 ( a b 2 + b c 2 + c a 2 − 3 a b c ) + 19 ( a 2 b + b 2 c + c 2 a − 3 a b c ) ≥ 0 \begin{align} &9(5a + 6b)(5b + 6c)(5c + 6a) - 11^3(a + b + c)(ab + bc + ca) \\ &=19 a^2 b+289 a b^2+289 a^2 c-924 a b c+19 b^2 c+19 a c^2+289 b c^2 \\ &=289(ab^2 + bc^2 + ca^2) + 19(a^2b + b^2c + c^2a) - 924abc \\ &=289(ab^2 + bc^2 + ca^2 - 3abc) + 19(a^2b + b^2c + c^2a - 3abc) \\ &\geq 0 \end{align} 9(5a+6b)(5b+6c)(5c+6a)113(a+b+c)(ab+bc+ca)=19a2b+289ab2+289a2c924abc+19b2c+19ac2+289bc2=289(ab2+bc2+ca2)+19(a2b+b2c+c2a)924abc=289(ab2+bc2+ca23abc)+19(a2b+b2c+c2a3abc)0 这就证明了(10)式成立。故而 1 1 3 ( a + b + c ) ( a b + b c + c a ) ( 5 a + 6 b ) ( 5 b + 6 c ) ( 5 c + 6 a ) ≤ 9 \frac {11^3 (a + b+ c) (ab + bc + ca)} {(5a + 6b)(5b + 6c)(5c + 6a)}\leq 9 (5a+6b)(5b+6c)(5c+6a)113(a+b+c)(ab+bc+ca)9,代入(8)式可得:
11 a 5 a + 6 b + 11 b 5 b + 6 c + 11 c 5 c + 6 a ≤ 3 \begin{align} \sqrt{\frac{11a}{5a+6b}}+\sqrt{\frac{11b}{5b+6c}}+\sqrt{\frac{11c}{5c+6a}} \leq 3 \end{align} 5a+6b11a +5b+6c11b +5c+6a11c 3 因此不等式(1)成立。 □ \quad \Box

证法二

我们证明一个比(1)式更一般的结论(假设 a , b , c , x , y a,b,c,x,y a,b,c,x,y 同时为正实数):
( x + y ) a x a + y b + ( x + y ) b x b + y c + ( x + y ) c x c + y a ≤ 3 \begin{align} \sqrt{\frac{(x+y)a}{xa+yb}}+\sqrt{\frac{(x+y)b}{xb+yc}}+\sqrt{\frac{(x+y)c}{xc+ya}} \leq 3 \end{align} xa+yb(x+y)a +xb+yc(x+y)b +xc+ya(x+y)c 3 不等式(19)在当 x / y x/y x/y 的比值满足下述范围
2 − 3 cos ⁡ ( π / 18 ) + 3 sin ⁡ ( π / 18 ) ≤ y x ≤ − 1 + 3 cos ⁡ ( π / 9 ) − 3 sin ⁡ ( π / 9 ) \begin{align} 2 - \sqrt{3} \cos(\pi/18) + 3 \sin(\pi/18) \leq \frac{y}{x} \leq -1 + 3 \cos(\pi/9) - \sqrt{3} \sin(\pi/9) \end{align} 23 cos(π/18)+3sin(π/18)xy1+3cos(π/9)3 sin(π/9) 时成立。上述比值范围改写成数值结果即为 0.81521 < y / x < 1.22668 0.81521<y/x<1.22668 0.81521<y/x<1.22668

不等式(19)中,取 x = 5 , y = 6 x=5, y=6 x=5,y=6 时即为(1),显然满足(20),故我们只需证明(19)式在(20)的条件下成立即可。

通过柯西·施瓦兹不等式,我们有:
( ( x + y ) a x a + y b + ( x + y ) b x b + y c + ( x + y ) c x c + y a ) 2 = ( x c + y a ( x + y ) a ( x c + y a ) ( x a + y b ) + x a + y b ( x + y ) b ( x a + y b ) ( x b + y c ) + x b + y c ( x + y ) c ( x b + y c ) ( x c + y a ) ) 2 ≤ ( x c + y a + x a + y b + x b + y c ) ( ( x + y ) a ( x c + y a ) ( x a + y b ) + ( x + y ) b ( x a + y b ) ( x b + y c ) + ( x + y ) c ( x b + y c ) ( x c + y a ) ) = ( x + y ) ( a + b + c ) ⋅ a b x 2 + 2 a b x y + a b y 2 + a c x 2 + 2 a c x y + a c y 2 + b c x 2 + 2 b c x y + b c y 2 ( x a + y b ) ( x c + y a ) ( x b + y c ) = ( x + y ) 3 ( a + b + c ) ( b c + a b + a c ) ( x a + y b ) ( x c + y a ) ( x b + y c ) \begin{align} &\left(\sqrt{\frac{(x+y)a}{xa+yb}}+\sqrt{\frac{(x+y)b}{xb+yc}}+\sqrt{\frac{(x+y)c}{xc+ya}}\right)^2\\ =&\Bigg(\Bigg. \sqrt{xc + ya}\sqrt{\frac{(x + y) a}{(x c + y a) (x a + y b)}} + \\ & \sqrt{xa + yb}\sqrt{\frac{(x + y) b}{(x a + y b) (x b + y c)}}+ \\ &\sqrt{xb + yc}\sqrt{\frac{(x + y) c}{(xb + yc) (x c + y a)}} \Bigg. \Bigg)^2 \\ \leq & \Bigg(\Bigg.x c + ya + xa + yb + xb + yc\Bigg. \Bigg)\Bigg(\Bigg.\frac{(x + y) a}{(x c + y a) (x a + y b)}+ \\ &\frac{(x + y) b}{(xa + y b) (x b + y c)}+\frac{(x + y) c}{(xb + yc) (xc + y a)}\Bigg. \Bigg) \\ =&(x+y) (a+b+c) \cdot \\ &\frac{a b x^2+2 a b x y+a b y^2+a c x^2+2 a c x y+a c y^2+b c x^2+2 b c x y+b c y^2}{(x a + y b) (x c+y a) (x b + y c)} \\ =&\frac{(x + y)^3 (a + b + c) (b c + a b + ac)}{(x a + y b) (x c+y a) (x b + y c)} \end{align} ===(xa+yb(x+y)a +xb+yc(x+y)b +xc+ya(x+y)c )2(xc+ya (xc+ya)(xa+yb)(x+y)a +xa+yb (xa+yb)(xb+yc)(x+y)b +xb+yc (xb+yc)(xc+ya)(x+y)c )2(xc+ya+xa+yb+xb+yc)((xc+ya)(xa+yb)(x+y)a+(xa+yb)(xb+yc)(x+y)b+(xb+yc)(xc+ya)(x+y)c)(x+y)(a+b+c)(xa+yb)(xc+ya)(xb+yc)abx2+2abxy+aby2+acx2+2acxy+acy2+bcx2+2bcxy+bcy2(xa+yb)(xc+ya)(xb+yc)(x+y)3(a+b+c)(bc+ab+ac) 其中的(28)式到(29)式是源于下面的因式分解:
a b x 2 + 2 a b x y + a b y 2 + a c x 2 + 2 a c x y + a c y 2 + b c x 2 + 2 b c x y + b c y 2 = ( x + y ) 2 ( a b + a c + b c ) \begin{align} & a b x^2+2 a b x y+a b y^2+a c x^2+2 a c x y+a c y^2+b c x^2+2 b c x y+b c y^2 \\ =& (x+y)^2 (a b+a c+b c) \end{align} =abx2+2abxy+aby2+acx2+2acxy+acy2+bcx2+2bcxy+bcy2(x+y)2(ab+ac+bc) 类似于证法一,我们下面来估计 ( x + y ) 3 ( a + b + c ) ( b c + a b + a c ) ( x a + y b ) ( x c + y a ) ( x b + y c ) \frac{(x + y)^3 (a + b + c) (b c + a b + ac)}{(x a + y b) (x c+y a) (x b + y c)} (xa+yb)(xc+ya)(xb+yc)(x+y)3(a+b+c)(bc+ab+ac) 的上界,不妨令:
( x + y ) 3 ( a + b + c ) ( b c + a b + a c ) ( x a + y b ) ( x c + y a ) ( x b + y c ) ≤ k \begin{align} \frac{(x + y)^3 (a + b + c) (b c + a b + ac)}{(x a + y b) (x c+y a) (x b + y c)} \leq k \end{align} (xa+yb)(xc+ya)(xb+yc)(x+y)3(a+b+c)(bc+ab+ac)k 又记 G = k ( x a + y b ) ( x c + y a ) ( x b + y c ) − ( x + y ) 3 ( a + b + c ) ( b c + a b + a c ) G=k (x a + y b) (x c+y a) (x b + y c)-(x + y)^3 (a + b + c) (b c + a b + ac) G=k(xa+yb)(xc+ya)(xb+yc)(x+y)3(a+b+c)(bc+ab+ac),下面则需要证明在 x , y x, y x,y 的某个条件下 G ≥ 0 G\geq 0 G0。我们改写一下 G G G
G = ( k x 2 y − ( x + y ) 3 ) ( b 2 c + c 2 a + a 2 b ) + ( k x y 2 − ( x + y ) 3 ) ( a b 2 + b c 2 + c a 2 ) + ( k x 3 + k y 3 − 3 ( x + y ) 3 ) a b c \begin{align} G=&(kx^2 y -(x+y)^3)(b^2 c+c^2a+a^2 b) \\ +&(kx y^2 -(x+y)^3)(ab^2+bc^2+ca^2) \\ +&(kx^3 + ky^3 -3 (x+y)^3)abc \end{align} G=++(kx2y(x+y)3)(b2c+c2a+a2b)(kxy2(x+y)3)(ab2+bc2+ca2)(kx3+ky33(x+y)3)abc 为验证上式,事实上由 G G G 的定义式展开而得:
G = ( a 2 b k x 2 y − a 2 b x 3 − 3 a 2 b x 2 y − 3 a 2 b x y 2 − a 2 b y 3 ) + ( a 2 c k x y 2 − a 2 c x 3 − 3 a 2 c x 2 y − 3 a 2 c x y 2 − a 2 c y 3 ) + ( a b 2 k x y 2 − a b 2 x 3 − 3 a b 2 x 2 y − 3 a b 2 x y 2 − a b 2 y 3 ) + ( a b c k x 3 + a b c k y 3 − 3 a b c x 3 − 9 a b c x 2 y − 9 a b c x y 2 − 3 a b c y 3 ) + ( a c 2 k x 2 y − a c 2 x 3 − 3 a c 2 x 2 y − 3 a c 2 x y 2 − a c 2 y 3 ) + ( b 2 c k x 2 y − b 2 c x 3 − 3 b 2 c x 2 y − 3 b 2 c x y 2 − b 2 c y 3 ) + ( b c 2 k x y 2 − b c 2 x 3 − 3 b c 2 x 2 y − 3 b c 2 x y 2 − b c 2 y 3 ) \begin{align} G = &(a^2 b k x^2 y-a^2 b x^3-3 a^2 b x^2 y-3 a^2 b x y^2-a^2 b y^3)+(a^2 c k x y^2-a^2 c x^3-\\ &3 a^2 c x^2 y-3 a^2 c x y^2-a^2 c y^3)+(a b^2 k x y^2-a b^2 x^3-3 a b^2 x^2 y-3 a b^2 x y^2-\\ &a b^2 y^3)+(a b c k x^3+a b c k y^3-3 a b c x^3-9 a b c x^2 y-9 a b c x y^2-3 a b c y^3)+\\ &(a c^2 k x^2 y-a c^2 x^3-3 a c^2 x^2 y-3 a c^2 x y^2-a c^2 y^3)+(b^2 c k x^2 y-b^2 c x^3-\\ &3 b^2 c x^2 y-3 b^2 c x y^2-b^2 c y^3)+(b c^2 k x y^2-b c^2 x^3-3 b c^2 x^2 y-\\ &3 b c^2 x y^2-b c^2 y^3) \end{align} G=(a2bkx2ya2bx33a2bx2y3a2bxy2a2by3)+(a2ckxy2a2cx33a2cx2y3a2cxy2a2cy3)+(ab2kxy2ab2x33ab2x2y3ab2xy2ab2y3)+(abckx3+abcky33abcx39abcx2y9abcxy23abcy3)+(ac2kx2yac2x33ac2x2y3ac2xy2ac2y3)+(b2ckx2yb2cx33b2cx2y3b2cxy2b2cy3)+(bc2kxy2bc2x33bc2x2y3bc2xy2bc2y3) 对上式关于 a 2 b , a 2 c , a b 2 , a b c , a c 2 , b 2 c , b c 2 a^2b,a^2c,ab^2,abc,ac^2,b^2c,bc^2 a2b,a2c,ab2,abc,ac2,b2c,bc2 合并同类项即可得到(33)至(35)式。

为了对 G G G 应用算术几何平均不等式后能继续将 G G G 放缩小,不妨假设(33)与(34)式中的 k x 2 y − ( x + y ) 3 ≥ 0 kx^2 y -(x+y)^3\geq 0 kx2y(x+y)30 以及 k x y 2 − ( x + y ) 3 ≥ 0 kx y^2 -(x+y)^3 \geq 0 kxy2(x+y)30。于是对(33)至(34)式关于 b 2 c + c 2 a + a 2 b b^2 c+c^2a+a^2 b b2c+c2a+a2b a b 2 + b c 2 + c a 2 ab^2+bc^2+ca^2 ab2+bc2+ca2 应用算术几何平均不等式得到:
G ≥ 3 ( ( k x 2 y − ( x + y ) 3 ) + ( k x y 2 − ( x + y ) 3 ) a b c + ( k x 3 + k y 3 − 3 ( x + y ) 3 ) a b c \begin{align} G\geq &3((kx^2y-(x+y)^3)+(kxy^2-(x+y)^3)abc+\\ &(kx^3 + ky^3 -3 (x+y)^3)abc \end{align} G3((kx2y(x+y)3)+(kxy2(x+y)3)abc+(kx3+ky33(x+y)3)abc 正如前面所述,我们期望 “在 x , y x, y x,y 的某个条件下 G ≥ 0 G\geq 0 G0”,不妨令上式不等式的右边等于零,则得:
3 ( k x 2 y − ( x + y ) 3 ) + 3 ( k x y 2 − ( x + y ) 3 ) = 3 ( x + y ) 3 − k x 3 − k y 3 \begin{align} 3(kx^2y-(x+y)^3)+3(kxy^2-(x+y)^3)=3(x+y)^3-kx^3 - ky^3 \end{align} 3(kx2y(x+y)3)+3(kxy2(x+y)3)=3(x+y)3kx3ky3 因为 3 ( k x 2 y − ( x + y ) 3 ) + 3 ( k x y 2 − ( x + y ) 3 ) = ( 3 k − 18 ) x 2 y + ( 3 k − 18 ) x y 2 − 6 ( x 3 + y 3 ) 3(kx^2y-(x+y)^3)+3(kxy^2-(x+y)^3)=(3 k-18) x^2 y+(3 k-18) x y^2-6 (x^3+y^3) 3(kx2y(x+y)3)+3(kxy2(x+y)3)=(3k18)x2y+(3k18)xy26(x3+y3),以及 3 ( x + y ) 3 − k x 3 − k y 3 = ( 3 − k ) x 3 + ( 3 − k ) y 3 + 9 x 2 y + 9 x y 2 3(x+y)^3-kx^3 - ky^3=(3-k) x^3+(3-k) y^3+9 x^2 y+9 x y^2 3(x+y)3kx3ky3=(3k)x3+(3k)y3+9x2y+9xy2,对比这里前后的系数项可得,当 k = 9 k=9 k=9 时,(44)式成立。即在下述假设下,
9 x 2 y − ( x + y ) 3 ≥ 0 9 x y 2 − ( x + y ) 3 ≥ 0 \begin{align} 9x^2 y -(x+y)^3\geq 0 \\ 9x y^2 -(x+y)^3 \geq 0 \end{align} 9x2y(x+y)309xy2(x+y)30 我们有:
( x + y ) 3 ( a + b + c ) ( b c + a b + a c ) ( x a + y b ) ( x c + y a ) ( x b + y c ) ≤ 9 \begin{align} \frac{(x + y)^3 (a + b + c) (b c + a b + ac)}{(x a + y b) (x c+y a) (x b + y c)} \leq 9 \end{align} (xa+yb)(xc+ya)(xb+yc)(x+y)3(a+b+c)(bc+ab+ac)9 恒成立。代入(29)式可得(19)式成立。下面我们来讨论 x , y x,y x,y 的取值范围,且注意到 x , y x,y x,y 同时为正。将(45)、(46)式变形为:
( x + y ) 3 x 2 y ≤ 9 ( x + y ) 3 x y 2 ≤ 9 \begin{align} \frac{(x+y)^3}{x^2y}\leq 9 \\ \frac{(x+y)^3}{xy^2}\leq 9 \end{align} x2y(x+y)39xy2(x+y)39 进一步化简为:
x y + 3 + 3 y x + ( y x ) 2 ≤ 9 ( x y ) 2 + 3 x y + 3 + y x ≤ 9 \begin{align} \frac{x}{y}+3+3\frac{y}{x}+\left(\frac{y}{x}\right)^2\leq 9 \\ \left(\frac{x}{y}\right)^2+3\frac{x}{y}+3+\frac{y}{x}\leq 9 \end{align} yx+3+3xy+(xy)29(yx)2+3yx+3+xy9 t = y / x t=y/x t=y/x,上式化简可得
{ t 3 + 3 t 2 − 6 t + 1 ≤ 0 ( a ) t 3 − 6 t 2 + 3 t + 1 ≤ 0 ( b ) \begin{cases} t^3+3t^2-6t+1\leq 0 \hspace{5mm} &(a)\\ t^3-6t^2+3t+1 \leq 0 & (b) \end{cases} {t3+3t26t+10t36t2+3t+10(a)(b) 因此条件(48)和(49)归结为三次不等式组(a)和(b)同时成立,并求出 t t t 的范围。解三次不等式组是一个比较复杂的问题,但我们可以求出三次方程(a)与(b)的对应 6 6 6 组实根。

(a)对应的三组实根为:
− 4.41147412780977... -4.41147412780977... 4.41147412780977... 0.184792530904095... 0.184792530904095... 0.184792530904095... 1.22668159690568... 1.22668159690568... 1.22668159690568...

(b)对应的三组实根为:
− 0.226681596905677... -0.226681596905677... 0.226681596905677... 0.815207469095905... 0.815207469095905... 0.815207469095905... 5.41147412780977... 5.41147412780977... 5.41147412780977...

我们先来看上面两个三次函数的图像:
在这里插入图片描述我们再把中间 [ − 1 , 2 ] [-1,2] [1,2] 的部分单独画出来:
在这里插入图片描述易知, t 3 + 3 t 2 − 6 t + 1 t^3+3t^2-6t+1 t3+3t26t+1 t < − 4.41147412780977... t< -4.41147412780977... t<4.41147412780977... 0.184792530904095... < t < 1.22668159690568... 0.184792530904095...< t < 1.22668159690568... 0.184792530904095...<t<1.22668159690568... 时为负,当 − 4.41147412780977... < t < 0.184792530904095... -4.41147412780977...< t < 0.184792530904095... 4.41147412780977...<t<0.184792530904095... t > 1.22668159690568... t > 1.22668159690568... t>1.22668159690568... 时为正。

同理, t 3 − 6 t 2 + 3 t + 1 t^3-6t^2+3t+1 t36t2+3t+1 t < − 0.226681596905677... t<-0.226681596905677... t<0.226681596905677... 0.815207469095905... < t < 5.41147412780977... 0.815207469095905...<t<5.41147412780977... 0.815207469095905...<t<5.41147412780977... 时为负,当 − 0.226681596905677... < t < 0.815207469095905... -0.226681596905677...<t<0.815207469095905... 0.226681596905677...<t<0.815207469095905... t > 5.41147412780977... t>5.41147412780977... t>5.41147412780977... 时为正。

x , y x,y x,y 同时为正知 t > 0 t>0 t>0,从而可得,当 0.815207469095905... < t < 1.22668159690568... 0.815207469095905...<t<1.22668159690568... 0.815207469095905...<t<1.22668159690568... ,上面的(a)(b)两式同时成立。 t < − 4.41147412780977... t<-4.41147412780977... t<4.41147412780977... 的结果舍去。

通过进一步的符号计算可得 t 3 − 6 t 2 + 3 t + 1 = 0 t^3-6t^2+3t+1=0 t36t2+3t+1=0 的一个根的精确表达式为:
0.815207469095905... = 2 − 3 cos ⁡ ( π / 18 ) + 3 sin ⁡ ( π / 18 ) 0.815207469095905...=2 - \sqrt{3} \cos(\pi/18) + 3 \sin(\pi/18) 0.815207469095905...=23 cos(π/18)+3sin(π/18) t 3 + 3 t 2 − 6 t + 1 = 0 t^3+3t^2-6t+1=0 t3+3t26t+1=0 的一个根的精确表达式为: 1.22668159690568... = − 1 + 3 cos ⁡ ( π / 9 ) − 3 sin ⁡ ( π / 9 ) 1.22668159690568...=-1 + 3 \cos(\pi/9) - \sqrt{3} \sin(\pi/9) 1.22668159690568...=1+3cos(π/9)3 sin(π/9) 最后我们把结论总结如下,对于正实数 a , b , c , x , y a,b,c,x,y a,b,c,x,y,不等式(19):
( x + y ) a x a + y b + ( x + y ) b x b + y c + ( x + y ) c x c + y a ≤ 3 \sqrt{\frac{(x+y)a}{xa+yb}}+\sqrt{\frac{(x+y)b}{xb+yc}}+\sqrt{\frac{(x+y)c}{xc+ya}} \leq 3 xa+yb(x+y)a +xb+yc(x+y)b +xc+ya(x+y)c 3 在当 2 − 3 cos ⁡ ( π / 18 ) + 3 sin ⁡ ( π / 18 ) ≤ y x ≤ − 1 + 3 cos ⁡ ( π / 9 ) − 3 sin ⁡ ( π / 9 ) 2 - \sqrt{3} \cos(\pi/18) + 3 \sin(\pi/18)\leq \frac{y}{x} \leq -1 + 3 \cos(\pi/9) - \sqrt{3} \sin(\pi/9) 23 cos(π/18)+3sin(π/18)xy1+3cos(π/9)3 sin(π/9) 时成立。

这就完成了最终证明。 □ \quad \Box

证法二的补充

特别地我们可以得到,当 x , y x,y x,y 为正整数时,若 x = y x=y x=y,则(19)成立。在 y > x y>x y>x时,若 y = x + 1 y=x+1 y=x+1,则当 x ≥ 5 x\geq 5 x5 时,(19)成立;若 y = x + 2 y=x+2 y=x+2,则当 x ≥ 9 x \geq 9 x9 时,(19)成立 ⋯ \cdots 。在 y < x y<x y<x 时,若 y = x − 1 y=x-1 y=x1,则当 x ≥ 6 x\geq 6 x6 时,(19)成立;若 y = x − 2 y=x-2 y=x2,则当 x ≥ 11 x\geq 11 x11 时,(19)成立 ⋯ \cdots


参考文献
[1] Inequality

本文来自互联网用户投稿,该文观点仅代表作者本人,不代表本站立场。本站仅提供信息存储空间服务,不拥有所有权,不承担相关法律责任。如若转载,请注明出处:http://www.mzph.cn/news/688046.shtml

如若内容造成侵权/违法违规/事实不符,请联系多彩编程网进行投诉反馈email:809451989@qq.com,一经查实,立即删除!

相关文章

leetcode hot100不同路径

本题可以采用动态规划来解决。还是按照五部曲来做 确定dp数组&#xff1a;dp[i][j]表示走到&#xff08;i&#xff0c;j&#xff09;有多少种路径 确定递推公式&#xff1a;我们这里&#xff0c;只有两个移动方向&#xff0c;比如说我移动到&#xff08;i&#xff0c;j&#x…

STM32 寄存器操作 systick 滴答定时器 与中断

一、什么是 SysTick SysTick—系统定时器是属于CM3内核中的一个外设&#xff0c;内嵌在NVIC中。系统定时器是一个24bit的向下递减的计数器&#xff0c; 计数器每计数一次的时间为1/SYSCLK&#xff0c;一般我们设置系统时钟SYSCLK等于72M。当重装载数值寄存器的值递减到0的时候…

python学习(三):pip安装及如何加速安装第三方组件

pip全称Package Installer for Python&#xff0c;即用来安装第三方组件的工具 一.安装pip Python3中setuptools、Pip安装详解 1、安装setuptools 命令如下&#xff1a; wget --no-check-certificate https://pypi.python.org/packages/source/s/setuptools/setuptools-19.…

C语言—指针

碎碎念:做指针题的时候我仿佛回到了原点&#xff0c;总觉得目的是为了把框架搭建起来&#xff0c;我胡说的哈31 1.利用指针变量将一个数组中的数据反向输出。 /*1.利用指针变量将一个数组中的数据反向输出。*/#include <stdio.h> #include <time.h> #include <…

phpstrom创建thinkphp项目

安装php和composer 参考 安装phpstrom 创建项目 查看thinkphp版本 https://packagist.org/packages/topthink/think 打开所在项目编辑配置 即可调试运行

二维码的颜色怎么改变?轻松3步修改二维码样式

怎么修改二维码的颜色呢&#xff1f;一般我们制作的二维码或者经过系统生成的二维码大多都是黑白颜色的&#xff0c;有些小伙伴会觉得不太美观无法满足自己的使用需求。那么对于想要修改二维码样式的小伙伴&#xff0c;可以使用二维码生成器的二维码美化功能来处理&#xff0c;…

07 按键控制 LED

按键简介 按键开关是一种电子开关&#xff0c;属于电子元器件类。常见的按键开关有两种&#xff0c;第一种是轻触式按键开关&#xff08;简称轻触开关&#xff09;&#xff0c;使用时以向开关的操作方向施加压力使内部电路闭合接通&#xff0c;当撤销压力时开关断开&#xff0…

git分布式版本控制工具基本操作

Windows操作 1.1 git基本操作 1.设置用户签名 git config user.name xx git config user.email xxb163.com2.初始化本地库 git init3.查看本地库状态 git status4.添加暂存区 git add 文件名称 git add *5.提交本地库 git commit -m "描述信息" 文件6.查看版本…

《富爸爸:巴比伦最富有的人》读书笔记

目录 作者简介 感悟 经典摘录 观点&#xff1a; 支付给自己(理解是投资自己) 观点&#xff1a;源源不断地放入金币 观点&#xff1a; 把收入的一部分留给我自己 观点&#xff1a; 从专业的人士得到建议 观点&#xff1a;一旦为自己规定了任务&#xff0c;就一定要完成 …

微波炉维修笔记

微波主要是靠2.45GHz 左右的微波(12.2cm 波长)加热水分子实现食物加热 所有不要使用金属器皿进行加热&#xff0c;要么因为电磁屏蔽&#xff0c;起不到加热效果&#xff0c;要么火光四射 微波炉基本组成 借鉴姜师傅的视频 碰到不加热其它都正常的问题 1.检查高压电容 使用万…

自动驾驶中的 DCU、MCU、MPU、SOC 和汽车电子架构

自动驾驶中的 DCU、MCU、MPU 1. 分布式电子电气架构2. 域集中电子电气架构架构2.1 通用硬件定义 3. 车辆集中电子电气架构4. ADAS/AD系统方案演变进程梳理4.1 L0-L2级别的ADAS方案4.2 L2以上级别的ADAS方案 5. MCU和MPU区别5.1 MCU和MPU的区别5.2 CPU与SoC的区别5.3 举个例子 R…

【STM32】软件SPI读写W25Q64芯片

目录 W25Q64模块 W25Q64芯片简介 硬件电路 W25Q64框图 Flash操作注意事项 状态寄存器 ​编辑 指令集 INSTRUCTIONS​编辑 ​编辑 SPI读写W25Q64代码 硬件接线图 MySPI.c MySPI.h W25Q64 W25Q64.c W25Q64.h main.c 测试 SPI通信&#xff08;W25Q64芯片简介&am…

说说对BOM的理解(常见的BOM对象了解哪些)

文章目录 一、是什么二、window三、location四、navigator五、screen六、history 一、是什么 BOM (Browser Object Model)&#xff0c;浏览器对象模型&#xff0c;提供了独立于内容与浏览器窗口进行交互的对象 其作用就是跟浏览器做一些交互效果,比如如何进行页面的后退&…

四川盐亭清代古寨重现,文物建筑保护引关注

近日&#xff0c;在四川盐亭的五台山深处&#xff0c;一处历经160余年风霜的古山寨遗迹重现天日。寨门巍峨&#xff0c;文字斑驳&#xff0c;诉说着清代同治年间的历史沧桑。然而&#xff0c;岁月侵蚀下&#xff0c;文物保护刻不容缓。温湿度波动等自然因素&#xff0c;对这些珍…

森林气象火险监测站

TH-SL10在广袤无垠的森林中&#xff0c;每一片树叶、每一缕风都蕴含着大自然的秘密。而在这片生机勃勃的绿色世界里&#xff0c;森林气象火险监测站就像是守护宝藏的“千里眼”和“顺风耳”&#xff0c;时刻警惕着潜在的危险。 一、森林气象火险监测站&#xff1a;实时监测的“…

生成式 AI - Diffusion 模型的数学原理(3)

来自 论文《 Denoising Diffusion Probabilistic Model》&#xff08;DDPM&#xff09; 论文链接&#xff1a; https://arxiv.org/abs/2006.11239 Hung-yi Lee 课件整理 文章目录 一、图像生成模型本质上的共同目标二、最大似然估计三、和VAE的关联四、概率计算 一、图像生成模…

GC9008 12V 全桥驱动芯片,可替代TMI8118,应用于摄像机、消费类产品上

GC9008 是一款 12V 全桥驱动芯片&#xff0c;为提供高性价比的方案。它能提供 0.1A 的持续输出电流。可以工作在 4.5~15V 的电源电压上。 具有 PWM&#xff08;IN1/IN2&#xff09;输入接口,与行业标准器件兼容.是 SOP8封装&#xff0c;GC9008D是DIP封装 芯片特点 ● H 桥电机…

计算机服务器中了_locked勒索病毒怎么办?Encrypted勒索病毒解密数据恢复

随着网络技术的不断发展&#xff0c;数字化办公已经成为企业生产运营的根本&#xff0c;对于企业来说&#xff0c;数据至关重要&#xff0c;但网络威胁无处不在&#xff0c;近期&#xff0c;云天数据恢复中心接到很多企业的求助&#xff0c;企业的计算机服务器遭到了_locked勒索…

Python 基于 AI 动物识别技术的研究与实现,附源码

博主介绍&#xff1a;✌程序员徐师兄、7年大厂程序员经历。全网粉丝30W、csdn博客专家、掘金/华为云/阿里云/InfoQ等平台优质作者、专注于Java技术领域和毕业项目实战✌ &#x1f345;文末获取源码联系&#x1f345; &#x1f447;&#x1f3fb; 精彩专栏推荐订阅&#x1f447;…

迁移SVN和GIT的云端数据

在新服务器搭建GIT仓库 教程很多&#xff0c;大致的流程是&#xff1a; 1. 新建linux用户密码专用于git操作 2. 新建git库的存放文件夹并在此初始化git 3. 配置git库所在目录权限 *只需要有一个库和有一个用户&#xff0c;与在windows上建库是一样的。不需要搭建类似gitla…